× Binary Unit Round-off Roundoff Error Number System with base \(\beta\) Floating point error analysis Orders of Convergence Newton's Method Error Analysis for Newton's Method Theorems on Newton's Method Horner's Algorithm Contractive Mapping Polynomial Interpretation Hermite Interpolation Change of Intervals Gaussian Quadrature Error Analysis for Gaussian Quadrature Composite Formulae \(\theta\)-Method Error Initial Value Problem Stability Local Truncation Error for Multistep Methods Schur's Form Hermite Matrix QR Factorization Householder LU Decomposition Norm Positive Definite Gerschgorin Theorem Rayleigh Quotient August 2017 August 2018 January 2019 August 2019 January 2020 August 2020 January 2021 August 2021 January 2022 August 2022 January 2023 August 2023 January 2024 References
☰ Menu

My name is Hanzhang Yin, and I have developed this website as a resource to facilitate the review of key concepts in abstract algebra, and it is not fully completed. Feel free to email me at hanyin@ku.edu if there are any errors or suggestions for improvement.

2020 (January) Numerical Analysis Qualifying Examination

1. (a) Write Newton’s iteration scheme for finding a root of an equation \( f(x) = 0 \).

(b) Suppose that \( f(x) \in C^m(a, b) \) for some integer \( m \geq 1 \) and \( p \in (a, b) \) is a root of \( f(x) = 0 \). Assume that the sequence \(\{x_n\} \subset (a, b) \) generated by Newton’s iteration converges to \( p \). Prove if \( f'(p) \ne 0 \), and if \( f''(p) = \ldots = f^{(m-1)}(p) = 0 \) when \( m \geq 3 \), then \[ \lim_{n \to \infty} \left| \frac{p - x_n}{(p - x_{n-1})^m} \right| = \frac{m - 1}{m!} \left| \frac{f^{(m)}(p)}{f'(p)} \right| \]

Solution (a). Newton's iteration scheme for finding a root of an equation \( f(x) = 0 \) is given by \[ x_{n+1} = x_n - \frac{f(x_n)}{f'(x_n)}, \] where \(x_0\) is close to the root of \(f(x) = 0\).

Solution (b). Suppose that \(p- x_{n-1} = e\). Since the sequence \(\{x_n\}\) generated by Newton's iteration converges to \(p\), we have \(e\to 0\) as \(n\to \infty\). Then we can have \[ \begin{align} f(x_{n-1}) &= f(p) + f'(p)(-e) + \frac{f''(p)}{2}(-e)^2 + \cdots + \frac{f^{(m-1)}(p)}{(m-1)!}(-e)^{m-1} + \frac{f^{(m)}(p)}{m!}(-e)^{m} + \mathcal{O}(e^{m+1}) \\ &= 0 + f'(p)(-e) + 0 + \cdots + 0 + \frac{f^{(m)}(p)}{m!}(-e)^{m} + \mathcal{O}(e^{m+1}) \\ &= f'(p)(-e) + \frac{f^{(m)}(p)}{m!}(-e)^{m} + \mathcal{O}(e^{m+1}). \\ f'(x_{n-1}) &= f'(p) + f''(p)(-e) + \cdots + \frac{f^{(m-1)}(p)}{(m-2)!}(-e)^{m-2} + \frac{f^{(m)}(p)}{(m-1)!}(-e)^{m-1} + \mathcal{O}(e^{m}) \\ &= f'(p) + \frac{f^{(m)}(p)}{(m-1)!}(-e)^{m-1} + \mathcal{O}(e^{m}), \end{align} \] for \(f''(p) = \cdots = f^{(m-1)}(p) = 0\). Then, we have \[ \begin{align} p - x_n &= p - x_{n-1} + \frac{f(x_{n-1})}{f'(x_{n-1})} \\ &= e + \dfrac{f'(p)(-e) + \frac{f^{(m)}(p)}{m!}(-e)^{m} + \mathcal{O}(e^{m+1})}{f'(p) + \frac{f^{(m)}(p)}{(m-1)!}(-e)^{m-1} + \mathcal{O}(e^{m})} \\ &= \dfrac{f'(p)e -\frac{f^{(m)}(p)}{(m-1)!}(-e)^{m} + \mathcal{O}(e^{m+1})}{f'(p) + \frac{f^{(m)}(p)}{(m-1)!}(-e)^{m-1} + \mathcal{O}(e^{m})} + \dfrac{f'(p)(-e) + \frac{f^{(m)}(p)}{m!}(-e)^{m} + \mathcal{O}(e^{m+1})}{f'(p) + \frac{f^{(m)}(p)}{(m-1)!}(-e)^{m-1} + \mathcal{O}(e^{m})} \\ &= \dfrac{ \frac{f^{(m)}(p)}{m!}(-e)^{m}-\frac{f^{(m)}(p)}{(m-1)!}(-e)^{m} + \mathcal{O}(e^{m+1})}{f'(p) + \frac{f^{(m)}(p)}{(m-1)!}(-e)^{m-1} + \mathcal{O}(e^{m})} \\ &= \dfrac{\left(\frac{f^{(m)}(p)}{m!}(-e)^{m}-\frac{f^{(m)}(p)}{(m-1)!}(-e)^{m} + \mathcal{O}(e^{m+1})\right)m!}{\left(f'(p) + \frac{f^{(m)}(p)}{(m-1)!}(-e)^{m-1} + \mathcal{O}(e^{m})\right)m!} \\ &= \dfrac{f^{(m)}(p)(-e)^{m}-mf^{(m)}(p)(-e)^{m} + \mathcal{O}(e^{m+1})}{f'(p)m! + mf^{(m)}(p)(-e)^{m-1} + \mathcal{O}(e^{m})} \\ \dfrac{p - x_{n}}{p - x_{n-1}} &= \dfrac{f^{(m)}(p)(-e)^{m}-mf^{(m)}(p)(-e)^{m} + \mathcal{O}(e^{m+1})}{f'(p)m! + mf^{(m)}(p)(-e)^{m-1} + \mathcal{O}(e^{m})}\cdot \dfrac{1}{e^m} \\ &= \dfrac{f^{(m)}(p)(-1)^{m}-mf^{(m)}(p)(-1)^{m} + \mathcal{O}(e)}{f'(p)m! + mf^{(m)}(p)(-e)^{m-1} + \mathcal{O}(e^{m})} \\ &= (-1)^{m}\dfrac{f^{(m)}(p)-mf^{(m)}(p) + \mathcal{O}(e)}{f'(p)m! + mf^{(m)}(p)(-e)^{m-1} + \mathcal{O}(e^{m})} \\ \end{align} \] Hence, we have \[ \left|\dfrac{p - x_{n}}{p - x_{n-1}}\right| = \left|\dfrac{f^{(m)}(p)-mf^{(m)}(p) + \mathcal{O}(e)}{f'(p)m! + mf^{(m)}(p)(-e)^{m-1} + \mathcal{O}(e^{m})}\right| = \dfrac{|f^{(m)}(p)-mf^{(m)}(p) + \mathcal{O}(e)|}{|f'(p)m! + mf^{(m)}(p)(-e)^{m-1} + \mathcal{O}(e^{m})|}. \] Since \(\lim_{n\to\infty} e = 0\), we have \[ \lim_{n\to\infty} \left|\dfrac{p - x_{n}}{p - x_{n-1}}\right| = \dfrac{|f^{(m)}(p)-mf^{(m)}(p)|}{|f'(p)m!|} = \dfrac{m - 1}{m!} \left| \dfrac{f^{(m)}(p)}{f'(p)} \right|. \tag*{\(\blacksquare\)} \]

Problem 4. Let \( A \in \mathbb{C}^{n \times n} \) be a non-singular matrix, and let \( 0 \neq b \in \mathbb{C}^n \) be a fixed vector. Suppose that you have an algorithm for solving \( Ax = b \) that produces an approximate solution \( \tilde{x} (\neq 0) \) that satisfies \[ (A + \Delta A) \tilde{x} = b, \] for some matrix \( \Delta A \) such that \[ \frac{\|\Delta A\|}{\|A\|} \leq \epsilon, \] with any consistent matrix norm \( \|\cdot\| \).

(a) Prove that for every \( \epsilon > 0 \) \[ \frac{\|\tilde{x} - x\|}{\|\tilde{x}\|} \leq \kappa(A) \epsilon, \] where \( x \in \mathbb{C}^n \) is the exact solution of \( Ax = b \), and \( \kappa(A) \) is the condition number of \( A \).

(b) Suppose that (a) holds asymptotically as \( \epsilon \to 0 \). Prove that \[ \frac{\|\tilde{x} - x\|}{\|x\|} \leq \kappa(A) \epsilon + \mathcal{O}(\epsilon^2), \quad \text{as} \quad \epsilon \to 0. \]

Proof. Given that \((A + \Delta A) \tilde{x} = b \) and \(A x = b\), we have \[ \begin{align} Ax - (A + \Delta A) \tilde{x} &= 0 \\ Ax - A\tilde{x} - \Delta A \tilde{x} &= 0 \\ A(x - \tilde{x}) &= \Delta A \tilde{x}. \end{align} \] Since \(A\) is non-singular, we have \[ x - \tilde{x} = A^{-1} \Delta A \tilde{x}. \] By Triangle Inequality, we have \[ \|x - \tilde{x}\| = \|A^{-1} \Delta A \tilde{x}\| \leq \|A^{-1}\| \|\Delta A\| \|\tilde{x}\|. \] Since \(A\) is non-singular and \(b\neq 0\), if \(x = 0\) then \(b = 0\) since \(A\cdot x = 0\), which is a contradiction. Thus, we can know that \(x\neq 0\), which implies that \(\|x\|>0\). Therefore, we have \[ \frac{\|x - \tilde{x}\|}{\|\tilde{x}\|} \leq \|A^{-1}\| \|\Delta A\|. \] Since \(\frac{\|\Delta A\|}{\|A\|} \leq \epsilon\), we have \[ \begin{align} \|\Delta A\| &\leq \epsilon \|A\| \\ \frac{\|x - \tilde{x}\|}{\|\tilde{x}\|} \leq \|A^{-1}\| \|\Delta A\| &\leq \|A^{-1}\| \|A\| \epsilon = \kappa(A) \epsilon. \tag*{\(\square\)} \end{align} \]


5. Let matrix \( A \in \mathbb{R}^{n \times n} \) be positive definite and matrix \( X \in \mathbb{R}^{n \times k} \) be of full column rank, i.e., \(\text{rank}(X) = k \).

  • (a) Write down a definition of the positive definite matrix.
  • (b) Prove that the matrix \( B = X^TAX, \, B \in \mathbb{R}^{k \times k} \) is positive definite.
  • (c) Prove that all principal sub-matrices of matrix \( A \) are positive definite. In particular, all the diagonal entries are positive.
  • (d) Prove that matrix \( A \) has an LU factorization and the diagonal entries of matrix \( U \) are all positive.

Solution (a). We say \(A\) is positive definite if for all \(x\in \mathbb{R}^n\setminus \{0\}\), we have \(x^T A x > 0\).

Proof (b). Let \(v\in \mathbb{R}^k\setminus \{0\}\). We denote \(X\) as \[ X = \begin{bmatrix} \vec{x_1} & \vec{x_2} & \cdots & \vec{x_k} \end{bmatrix}, \] where \(x_i\in \mathbb{R}^n\). Since \(X\) is of full column rank, we have \(\vec{x_1}, \vec{x_2}, \ldots, \vec{x_k}\) are linearly independent. Now, let \(v = \begin{bmatrix} v_1 & v_2 & \cdots & v_k \end{bmatrix}^T\). Then, we have \[ v^T\cdot X^T = \begin{bmatrix} v_1 & v_2 & \cdots & v_k \end{bmatrix} \begin{bmatrix} \vec{x_1}^T \\ \vec{x_2}^T \\ \vdots \\ \vec{x_k}^T \end{bmatrix} = v_1 \vec{x_1} + v_2 \vec{x_2} + \cdots + v_k \vec{x_k}. \] Since \(\{ \vec{x_1}, \vec{x_2}, \ldots, \vec{x_k} \}\) are linearly independent, we have \[ a_1\vec{x_1} + a_2 \vec{x_2} + \cdots + a_k \vec{x_k} = 0 \quad \text{if and only if} \quad a_1 = a_2 = \cdots = a_k = 0. \] And we know that there exists some \(v_i\neq 0\). Therefore, we have \[ v^T\cdot X^T \neq 0,\text{ and } (Xv)^T\neq 0, \] which implies that \(Xv\neq 0\). Now, we denote \(u = Xv\). Since \(A\) is positive definite and \(u\neq 0\), we have \[ v^T\cdot X^TAXv = (Xv)^TA(Xv) = u^TAu > 0. \tag*{\(\square\)} \]

Proof. According to the \(LU\) factorization, we can know that the diagonal entries of \(L\) are all ones. We denote \(L_k, U_k\) as the \(k\)th leading principal sub-matrix of \(L, U\), respectively. And we can know that \(A_k = L_kU_k\). Since \(\det A_k>0\) and \(\det L_k = 1\) and \(\det A_k = \det U_k \det L_k\), we can know that \(\det U_k > 0\). Since \(\det U_k = u_{11} \dots u_{kk}\), we can know that \(u_{11}\cdots u_{kk} > 0\). Now, we want to prove that \(u_{ii}>0\) by induction. Since \(\det U_1 > 0\), we can know that \(u_{11} > 0\). Suppose that \(u_{11}\cdots u_{ii} > 0\). Then, we have \(\det U_{i+1} = u_{11}\cdots u_{ii}u_{i+1,i+1} > 0\). It forces that \(u_{i+1,i+1} > 0\). Therefore, we can know that the diagonal elements of \(U\) are positive. Thus, \(D = \text{diag}(\sqrt{u_{11}}, \dots, \sqrt{u_{mm}})\) exists and is non-singular. Let \(R = D^{-1}U\). Since \(D^{-1} = \text{diag}(\frac{1}{\sqrt{u_{11}}}, \dots, \frac{1}{\sqrt{u_{mm}}})\), we can know that \(D^{-1}\) is an upper triangular matrix. Since \(U\) is an upper triangular matrix, we can know that \(R\), which is a product of two upper triangular matrices, is an upper triangular matrix. Now, we want to show that \(A = R^*R\). Since \(A\) is Hermitian, we have \(A = A^*\). Let \(A = LU\), then \(A = (LU)^* = U^*L^* = LU\). Since the diagonal entries of \(L\) are \(U\) are positive, we can know that \(L, U\) are non-singular, which implies that \(U^*, L^*\) are also non-singular. Then, given \(U^*L^* = LU\), we can know that \(U(L^*)^{-1} = L^{-1}U^*\). Again, according to the properties of upper and lower triangular matrices, we can know that \((L^*)^{-1}\) is an upper triangular matrix and \( U^*\) is a lower triangular matrix. Hence, \(U(L^*)^{-1}\) is an upper triangular matrix and \(L^{-1}U^*\) is a lower triangular matrix. Since \(U(L^*)^{-1} = L^{-1}U^*\), we can know that \(U(L^*)^{-1} = L^{-1}U^*\) is a diagonal matrix. Now, we look at the entries of \(L^{-1}U^*\). Firstly, the diagonal entries of \(L^{-1}\) are still ones and the diagonal entries of \(U^*\) are the same as the diagonal entries of \(U\). Now, by the matrix multiplication, we see that the diagonal entries of \(L^{-1}U^*\) are \(u_{11}, \dots, u_{mm}\). Since itself is a diagonal matrix, we can know that \[ L^{-1}U^* = \text{diag}(u_{11}, \dots, u_{mm}) = D^2. \] It shows that \(U^* = LD^2\). Given \(D^{-1}\) is a diagonal matrix, we can know that \((D^{-1})^* = D^{-1}\). \begin{align*} (D^{-1}U)^* &= U^*(D^{-1})^* = U^*D^{-1} = LD^2D^{-1} = LD. \end{align*} Therefore, we can see that \[ R^* R = (D^{-1}U)^*(D^{-1}U) = LDD^{-1}U = LU = A. \tag*{\(\square\)} \]